Limit of a sequence (3)

Calculus Level 5

If a ( x , y ) = 1 + x 2 y 2 , x N , y N a(x,y)=1+\dfrac{x^2}{y^2}, \ x \in \mathbb{N}, \ y \in \mathbb{N} then find the value of lim n k = 1 n a ( k , n ) n \lim_{n \rightarrow \infty} \prod_{k=1}^{n} \sqrt[n]{a(k,n)}

Details and Assumptions

r = 1 n a r = a 1 a 2 a 3 . . . . . a n \bullet \ \ \displaystyle\prod_{r=1}^{n} a_r=a_1 \cdot a_2 \cdot a_3 \ ..... \ a_n

Also try limit as a sequence (1) and (2) .


The answer is 1.302.

This section requires Javascript.
You are seeing this because something didn't load right. We suggest you, (a) try refreshing the page, (b) enabling javascript if it is disabled on your browser and, finally, (c) loading the non-javascript version of this page . We're sorry about the hassle.

1 solution

Pratik Shastri
Sep 10, 2014

Let the limit we need to find be L L .

L = lim n k = 1 n a ( k , n ) n = lim n k = 1 n 1 + k 2 n 2 n = lim n ( 1 + 1 2 n 2 ) ( 1 + 2 2 n 2 ) . . . . ( 1 + n 2 n 2 ) n \begin{aligned} L &=\lim_{n \rightarrow \infty} \prod_{k=1}^{n} \sqrt[n]{a(k,n)} \\ &=\lim_{n \rightarrow \infty} \prod_{k=1}^{n} \sqrt[n]{1+\dfrac{k^2}{n^2}} \\ &=\lim_{n \rightarrow \infty} \sqrt[n]{\left(1+\dfrac{1^2}{n^2}\right) \left(1+\dfrac{2^2}{n^2}\right) \ .... \ \left(1+\dfrac{n^2}{n^2}\right)} \end{aligned}

Taking natural logarithm on both the sides to convert the limit in question into a sum,

log L = lim n 1 n log ( ( 1 + 1 2 n 2 ) ( 1 + 2 2 n 2 ) . . . . ( 1 + n 2 n 2 ) ) = lim n 1 n r = 1 n log ( 1 + r 2 n 2 ) = lim n 1 / n n / n log ( 1 + x 2 ) d x = 0 1 log ( 1 + x 2 ) d x L = e 0 1 log ( 1 + x 2 ) d x \begin{aligned} \log L &=\lim_{n \rightarrow \infty} \dfrac{1}{n} \log{\left(\left(1+\dfrac{1^2}{n^2}\right) \left(1+\dfrac{2^2}{n^2}\right) \ .... \ \left(1+\dfrac{n^2}{n^2}\right)\right)} \\ &=\lim_{n \rightarrow \infty} \dfrac{1}{n} \displaystyle\sum_{r=1}^{n} \log {\left(1+\dfrac{r^2}{n^2}\right)} \\ &=\lim_{n \rightarrow \infty} \displaystyle\int_{1/n}^{n/n} \log (1+x^2) \ dx \\ &= \displaystyle\int_{0}^{1} \log {(1+x^2)} \ dx \\ \therefore L &=e^{\int_{0}^{1} \log {(1+x^2)} \ dx} \end{aligned}

Now to find the integral, we use IBP \rightarrow

0 1 log ( 1 + x 2 ) d x = x log ( 1 + x 2 ) 0 1 0 1 x 2 x 1 + x 2 d x = log 2 0 1 2 2 1 + x 2 d x = log 2 2 x 2 tan 1 x 0 1 = log 2 + π 2 2 \begin{aligned} \displaystyle\int_{0}^{1} \log {(1+x^2)} \ dx &= \left. x\log {(1+x^2)} \right|_{0}^{1}-\displaystyle\int_{0}^{1} x \dfrac{2x}{1+x^2} dx \\ &= \log 2-\displaystyle\int_{0}^{1} 2-\dfrac{2}{1+x^2} dx \\ &= \log 2 - \left. 2x-2\tan^{-1} x\right|_{0}^{1}=\log 2+\dfrac{\pi}{2}-2 \end{aligned}

So, L = 2 e π / 2 2 1.302 L=\boxed{2e^{\pi/2-2}} \approx \boxed{1.302}

@Anish Kelkar You can take limits separately on terms in multiplication only if the number of terms is finite .. Here the number of terms is dependent on n n .

Pratik Shastri - 6 years, 9 months ago

l i m n ( r = 1 n 1 + r 2 n 2 ) 1 / n = l i m n ( 1 + 1 n 2 ) 1 / n l i m n ( 1 + 2 n 2 ) 1 / n l i m n ( 1 + 3 n 2 ) 1 / n l i m n ( 1 + 4 n 2 ) 1 / n . . . . . . . . l i m n ( 1 + n 2 n 2 ) 1 / n = l i m n e 1 / n 3 e 2 2 / n 3 e 3 2 / n 3 e 4 2 / n 3 . . . . . . . . . . . . . e n 2 / n 3 = l i m n e n ( n + 1 ) ( 2 n + 1 ) 6 n 3 = e 1 3 \underset { n\rightarrow \infty }{ lim } \quad { \left( \prod _{ r=1 }^{ n }{ 1+\frac { { r }^{ 2 } }{ { n }^{ 2 } } } \right) }^{ 1/n }=\underset { n\rightarrow \infty }{ lim } \quad { \left( 1+\frac { 1 }{ n^{ 2 } } \right) }^{ 1/n }\quad \cdot \underset { n\rightarrow \infty }{ lim } \quad { \left( 1+\frac { 2 }{ n^{ 2 } } \right) }^{ 1/n }\underset { n\rightarrow \infty }{ lim } \quad { \left( 1+\frac { 3 }{ n^{ 2 } } \right) }^{ 1/n }\quad \cdot \underset { n\rightarrow \infty }{ lim } \quad { \left( 1+\frac { 4 }{ n^{ 2 } } \right) }^{ 1/n }\quad ........\underset { n\rightarrow \infty }{ lim } \quad { \left( 1+\frac { n^{ 2 } }{ n^{ 2 } } \right) }^{ 1/n }\quad \cdot \\ =\underset { n\rightarrow \infty }{ lim } \quad { e }^{ 1/n^{ 3 } }\cdot \quad { e }^{ 2^{ 2 }/n^{ 3 } }\cdot \quad { e }^{ 3^{ 2 }/n^{ 3 } }\cdot \quad { e }^{ 4^{ 2 }/n^{ 3 } }\cdot .............\quad \quad { e }^{ n^{ 2 }/n^{ 3 } }\cdot \\ =\underset { n\rightarrow \infty }{ lim } \quad { e }^{ \frac { n(n+1)(2n+1) }{ 6 } { n }^{ 3 } }\quad =\quad { e }^{ \frac { 1 }{ 3 } }

Anish Kelkar - 6 years, 9 months ago

Log in to reply

Sir I have some problem in ur second line.....ok n is a variable mind it...if it is a constant it was perfectly ok...see this stuff u can write x^2=x+x+....+x....differentiate both sides u get 2x=x...so where is the fallacy?It is that x is a variable no. so u can not apply that theorem that d(cx)/dx=c where c is a constant.Ok sir this is what I think I may be completely wrong if u have anything to say pls kindly do.Thanks.

rajdeep brahma - 3 years ago

@Anish Kelkar You cannot take the limit individually on the brackets like that

Pratik Shastri - 6 years, 9 months ago

Same as I did it.

Ronak Agarwal - 6 years, 9 months ago

Did you forget to add this to the problem statement ? -- " This too is a standard problem in most calculus textbooks for JEE(Advanced) " Just kidding ! : P .

Arif Ahmed - 6 years, 8 months ago

How did you convert that sum into the integral?

Bogdan Simeonov - 6 years, 9 months ago

Log in to reply

Using the concept of Riemann sum lim n 1 n r = f ( n ) g ( n ) h ( r n ) = lim n f ( n ) / n g ( n ) / n h ( x ) d x if lim n f ( n ) n is finite \lim_{n \rightarrow \infty} \dfrac{1}{n} \displaystyle\sum_{r=f(n)}^{g(n)} h \left(\dfrac{r}{n}\right)=\lim_{n \rightarrow \infty} \displaystyle\int_{f(n)/n}^{g(n)/n} h(x) \ dx \\ \text{if} \ \lim_{n \rightarrow \infty} \dfrac{f(n)}{n} \ \text{is finite}

Pratik Shastri - 6 years, 9 months ago

well if we simply solve it using limits . answer comes out to be e^(1/3)

Anish Kelkar - 6 years, 9 months ago

Well I donot think any mistake in it as it is a well defined property of limit of product of functions . ( NCERT Pg 292 Class XI)

Anish Kelkar - 6 years, 9 months ago

I am unable to understand where my mistake is ??

Anish Kelkar - 6 years, 9 months ago

yeah ! did same :) but it's bit over-rated .

A Former Brilliant Member - 4 years, 2 months ago

Easy problem dude

Rupayan Jana - 1 year, 10 months ago

0 pending reports

×

Problem Loading...

Note Loading...

Set Loading...